Fourier series of this scale and shift of triangle wave

  • Thread starter Thread starter zenterix
  • Start date Start date
  • Tags Tags
    Fourier series
Click For Summary
SUMMARY

The discussion centers on the Fourier series representation of a shifted triangle wave function, specifically the function defined as ##g(t) = tr\left(t + \frac{\pi}{2}\right) - \frac{\pi}{2}##. The triangle wave function ##tr(t)## is expressed as a series involving only odd integers, correcting an initial oversight in the summation. The final corrected Fourier series for ##g(t)## includes only sine terms due to the nature of the odd function, leading to the conclusion that the series simplifies to a sine series involving odd harmonics.

PREREQUISITES
  • Understanding of Fourier series and their applications
  • Familiarity with triangle wave functions
  • Knowledge of trigonometric identities and properties
  • Basic calculus and series convergence concepts
NEXT STEPS
  • Study the derivation of Fourier series for periodic functions
  • Explore the properties of odd and even functions in Fourier analysis
  • Learn about convergence criteria for Fourier series
  • Investigate the implications of shifting functions in Fourier series representation
USEFUL FOR

Mathematicians, engineers, and students studying signal processing or harmonic analysis who seek to deepen their understanding of Fourier series and their applications to wave functions.

zenterix
Messages
774
Reaction score
84
Homework Statement
Consider the periodic function of period ##2\pi##
Relevant Equations
$$g(t)=\begin{cases} t\ \ \ \ \ -\frac{\pi}{2}<t<\frac{\pi}{2} \\ \pi-t\ \ \ \ \ \frac{\pi}{2}<t<\frac{3\pi}{2} \end{cases}\tag{1}$$
This function seems to be ##tr\left (t+\frac{\pi}{2}\right )-\frac{\pi}{2}## where ##tr(t)## is the triangle wave function.

$$tr(t)=|t|\ \ \ \ \ -\frac{\pi}{2}<t<\frac{\pi}{2}\tag{2}$$

1710150127225.png


##tr## has Fourier series

$$tr(t)=\frac{\pi}{2}-\frac{4}{\pi}\sum\limits_{n=1}^\infty \frac{\cos{(nt)}}{n^2}\tag{2a}$$

Thus

$$g(t)=tr\left (t+\frac{\pi}{2}\right )-\frac{\pi}{2}\tag{3}$$

$$-\frac{4}{\pi}\sum\limits_{n=1}^\infty \frac{\cos{\left (n\left (t+\frac{\pi}{2}\right ) \right )}}{n^2}\tag{4}$$

Now

$$\cos{\left ( nt +n\frac{\pi}{2} \right )}=\cos{(nt)}\cos{\left (\frac{n\pi}{2}\right )}-\sin{(nt)}\sin{\left (\frac{n\pi}{2}\right )}\tag{5}$$

$$\sum\limits_{n=1}^\infty \cos{\left ( nt +n\frac{\pi}{2} \right )} = (-\sin{t})+(-\cos{2t})+(\sin{3t})+(\cos{4t})+(-\sin{5t})+(-\cos{6t})+\ldots\tag{6}$$

$$=\sum\limits_{k=0}^\infty (-1)^{k+1}\sin{(2k+1)t}+\sum\limits_{k=1}^\infty (-1)^k \cos{(2kt)}\tag{7}$$

If we sub this expression into (4) then

$$g(t)=-\frac{4}{\pi}\left ( \sum\limits_{k=0}^\infty (-1)^{k+1}\frac{\sin{(2k+1)t}}{(2k+1)^2}+\sum\limits_{k=1}^\infty (-1)^k \frac{\cos{(2kt)}}{(2k)^2} \right )\tag{8}$$

If the calculations are correct so far, my question is why this Fourier series has both cosine and sine terms when ##g## is odd.
 
Last edited:
Physics news on Phys.org
I think I need to take a break from studying today.

The answer to my question is a silly mistake.

Equation (2a) is incorrect. The summation is only over odd ##n##

$$tr(t)=\frac{\pi}{2}-\frac{4}{\pi}\sum\limits_{n\ \text{odd}} \frac{\cos{(nt)}}{n^2}\tag{2a}$$

Thus, equation (4) is also only over odd ##n##

$$-\frac{4}{\pi}\sum\limits_{n\ \text{odd}}^\infty \frac{\cos{\left (n\left (t+\frac{\pi}{2}\right ) \right )}}{n^2}\tag{4}$$

and in (5), since ##n## is odd then the first term on the rhs is zero

$$\cos{\left ( nt +n\frac{\pi}{2} \right )}=-\sin{(nt)}\sin{\left (\frac{n\pi}{2}\right )}\tag{5}$$

$$=\sum\limits_{k=0}^\infty (-1)^{k+1} \sin{((2k+1)t)}\tag{7}$$

Hence

$$g(t)=-\frac{4}{\pi}\left ( \sum\limits_{k=0}^\infty (-1)^{k+1}\frac{\sin{(2k+1)t}}{(2k+1)^2} \right )\tag{8}$$
 
  • Like
Likes FactChecker
Question: A clock's minute hand has length 4 and its hour hand has length 3. What is the distance between the tips at the moment when it is increasing most rapidly?(Putnam Exam Question) Answer: Making assumption that both the hands moves at constant angular velocities, the answer is ## \sqrt{7} .## But don't you think this assumption is somewhat doubtful and wrong?

Similar threads

  • · Replies 3 ·
Replies
3
Views
2K
  • · Replies 14 ·
Replies
14
Views
2K
  • · Replies 1 ·
Replies
1
Views
1K
Replies
6
Views
1K
  • · Replies 16 ·
Replies
16
Views
2K
  • · Replies 11 ·
Replies
11
Views
3K
  • · Replies 6 ·
Replies
6
Views
2K
  • · Replies 7 ·
Replies
7
Views
2K
  • · Replies 1 ·
Replies
1
Views
2K
  • · Replies 2 ·
Replies
2
Views
2K